Question

4. Golden Rule. Suppose that we have a standard Solow Model. There is no population or...

4. Golden Rule. Suppose that we have a standard Solow Model. There is no population or technology growth. (a) The rm problem is to maximize prots, t. The rm's problem can be written as: max KtC0;NtC0 t = AK t N1− t − wtNt − rtKt: The rm takes the factor prices as given. Find the rst order conditions characterizing the optimal rm behavior. (b) Use the FONCs from 4a to show that wtNt~Yt = 1 − , where Yt = AK t N1− t . (c) The household is endowed with labor and owns the capital stock and leases it to rms on a period-by- period basis. Firms remit any prots back to households. The household budget constraint is given by. Ct + It = wtNt + rtKt + t: Use your previous answers to show that the right hand side of the equation reduces to Yt = AK t N1− t . (d) Capital accumulation is standard: Kt+1 = (1 − )Kt + It where Kt is given in period t, as it is inherited from past decisions. Assume that the household consumes a constant fraction of its income each period, 1 − s, and supplies labor inelastically. Re-write the capital accumulation equation as a dierence equation relating kt+1 to kt and exogenous variables and parameters only, where k represents the per-worker level of capital. (e) Create a graph plotting kt+1 against kt. Argue that exists a steady state, k, at which kt+1 = kt. (f) Algebraically solve for the steady state stock, k, as well as steady state output, y, and consumption, c. (g) What value of s would maximize y? Do you think the household would like this saving rate? Why or why not? (h) What value of s would maximize current ct? Do you think it would be a good thing to have this saving rate? (i) What value of s would maximize steady state consumption, c? Please derive an analytical expression for the s that makes c as big as possible. (j) A reasonable value for is 0.33. With a saving rate between 10{15 percent, would the US be near the \Golden Rule" saving rate you found in (i)? If not, would you necessarily recommend that we increase our saving rate?

Homework Answers

Know the answer?
Your Answer:

Post as a guest

Your Name:

What's your source?

Earn Coins

Coins can be redeemed for fabulous gifts.

Not the answer you're looking for?
Ask your own homework help question
Similar Questions
Consider how unemployment would affect the Solow growth model. Suppose that output is produced according to...
Consider how unemployment would affect the Solow growth model. Suppose that output is produced according to the production function Y = Kα [(1 – u)L]1-α where K is capital, L is the labor force, and u is the natural rate of unemployment. The national saving rate is s, the labor force grows at rate n, and capital depreciates at rate δ. a. Write a condition that describes the golden rule steady state of this economy. b. Express the golden rule...
Consider a version of the Solow model where population grows at the constant rate ? >...
Consider a version of the Solow model where population grows at the constant rate ? > 0 and labour efficiency grows at rate ?. Capital depreciates at rate ? each period and a fraction ? of income is invested in physical capital every period. Assume that the production function is given by: ?t = ?ta(?t?t )1-a Where ??(0,1), ?t is output, ?t is capital, ?t is labour and ?t is labour efficiency. a. Show that the production function exhibits constant...
3- Growth Model Suppose that the output (Y) in the economy is given by the following...
3- Growth Model Suppose that the output (Y) in the economy is given by the following aggregate production function. Yt = Kt +Nt where the Kt is capital and Nt is population. Furthermore assume that the capital depreciate at the rate of ẟ and That saving constant and proportion s of income you may assume that ẟ>s 1-suppose that the population remains constant . solve for the steady state level of capital per worker 2- now suppose that the population...
(Neoclassical Growth Model). Consider the production function f(k) = Ak0.25, with A = 1, the saving...
(Neoclassical Growth Model). Consider the production function f(k) = Ak0.25, with A = 1, the saving rate s = 0.25, and the depreciation and population growth rates rates d = 0.15 and n = 0.10. The steady state level of capital per capita is k* = 1. For k0 = 0.5 and k0 = 1.5, as initial capital per capita, ll the values of per capita capital, output, the MPK, savings, required investment and the net capital accumulated (△k) in...
Consider a numerical example using the Solow growth model: The production technology is Y=F(K,N)=K0.5N0.5 and people...
Consider a numerical example using the Solow growth model: The production technology is Y=F(K,N)=K0.5N0.5 and people consume after saving a proportion of income, C=(1-s)Y. The capital per worker, k=K/N, evolves by (1+n)k’=szf(k)+(1-d)k. (a) Describe the steady state k* as a function of other variables. (b) Suppose that there are two countries with the same steady state capital per worker k* and zero growth rate of population(n=0), but differ by saving rate, s and depreciation rate, d. So we assume that...
2. Consider a numerical example using the Solow growth model: The production technology is Y=F(K,N)=K0.5N0.5 and...
2. Consider a numerical example using the Solow growth model: The production technology is Y=F(K,N)=K0.5N0.5 and people consume after saving a proportion of income, C=(1-s)Y. The capital per worker, k=K/N, evolves by (1+n)k’=szf(k)+(1-d)k. (a) Describe the steady state k* as a function of other variables (b) Suppose that there are two countries with the same steady state capital per worker k* and zero growth rate of population(n=0), but differ by saving rate, s and depreciation rate, d. So we assume...
Let’s solve the two sector model from page 281 of your textbook. The economy has two...
Let’s solve the two sector model from page 281 of your textbook. The economy has two sectors, manufacturing firms and research universities. The two sectors are described by the production functions Y = K1/2[(1-u)LE]1/2 ?E = u E where u is the fraction of labour force in universities (assume u is exogenous). Write the equation of motion of capital, ?K = sY - ?K, in intensive form. Write down the steady state condition and find the steady state level of...
Answer the following questions using the basic Solow growth model, without population growth or technological progress....
Answer the following questions using the basic Solow growth model, without population growth or technological progress. (a) Draw a diagram with per worker output, y, consumption, c, saving, s and investment, i, on the vertical axis and capital per worker, k, on the horizontal condition. On this diagram, clearly indicate steady-state values for c, i, and y. Briefly outline the condition that holds in the steady- state (i.e. what is the relationship between investment and the depreciation of capital?). (b)...
Assume that an economy described by the Solow model has the production function Y = K...
Assume that an economy described by the Solow model has the production function Y = K 0.4 ( L E ) 0.6, where all the variables are defined as in class. The saving rate is 30%, the capital depreciation rate is 3%, the population growth rate is 2%, and the rate of change in labor effectiveness (E) is 1%. For this country, what is f(k)? How did you define lower case k? Write down the equation of motion for k....
Consider the Solow grow model. Suppose for each unit of savings, the government consumes a fraction...
Consider the Solow grow model. Suppose for each unit of savings, the government consumes a fraction τ , so only the fraction 1 − τ would accumulate the capital stock. In other words, the law of motion for capital becomes: K1= (1 − δ)K + (1 − τ )sY where δ is the depreciation rate, s is the saving rate, and Y is aggregate output. Suppose production function is Y = zF(K, N). Follow the same steps we did in...
ADVERTISEMENT
Need Online Homework Help?

Get Answers For Free
Most questions answered within 1 hours.

Ask a Question
ADVERTISEMENT